Questions tagged [prime-numbers]

Questions where prime numbers play a key-role, such as: questions on the distribution of prime numbers (twin primes, gaps between primes, Hardy–Littlewood conjectures, etc); questions on prime numbers with special properties (Wieferich prime, Wolstenholme prime, etc.). This tag is often used as a specialized tag in combination with the top-level tag nt.number-theory and (if applicable) analytic-number-theory.

Filter by
Sorted by
Tagged with
2 votes
1 answer
290 views

Sums over primes in arithmetic progressions

Do we know anything about sums over primes in arithmetic progressions like the following: $$\sum_{\substack{q \equiv a (\text{mod } l) \\ q \le x}} q^{\alpha}$$ where $q$ is a prime and $\alpha > ...
Iguana's user avatar
  • 301
1 vote
0 answers
314 views

From an inequality for the Euler's totient function to a combination of Firoozbakht's conjecture and Nicolas' criterion for the Riemann hypothesis

In this post we ask about the veracity of an inequality deduced from a combination of Firoozbakht's conjecture (see [1] or [2]) and Nicolas' criterion for the Riemann hypothesis (see for instance [3])....
user142929's user avatar
3 votes
2 answers
216 views

The graph and sign of $p_n-\operatorname{ali}(n)$, where $p_n$ is the $n$-th prime and $\operatorname{ali}(n)$ the inverse of the logarithmic integral

I'm inspired in [1] to ask the following question. My problem is that I have not an implementation of the inverse of the logarithmic integral $\operatorname{Li}(x)=\int_2^x\frac{dt}{\log t}$, that ...
user142929's user avatar
4 votes
1 answer
282 views

A similar lemma to a lemma due to Lagarias, for the partial sums of reciprocal of primes

I was inspired in Lemma 3.1 of [1] and in the Theorem 4.12 of [2] to ask about a similar statement that shows Lagarias in his paper as Lemma 3.1. The Lemma from Lagarias's paper is that if $H(n)=\...
user142929's user avatar
2 votes
1 answer
281 views

Question on odd positive integers and Fermat factors

Let $n$ be an odd positive integer, Let $o=\operatorname{ord}_n 2$ be the order of 2 modulo $n$ and $m$ the period of $1/n, k$ is number of distinct odd residues contained in set $\{2^1,2^2,...,2^{n−1}...
Mike's user avatar
  • 359
4 votes
2 answers
438 views

Sharp estimates for Meissel-Mertens constant

I wondered if it is possible to get a similar inequality like $(1.1)$ of Michael D. Hirschhorn, Approximating Euler's Constant, The Fibonacci Quarterly, Volume 49, Number 3 (August 2011) for the ...
user142929's user avatar
0 votes
1 answer
218 views

Is this theorem on the abundance of prime patterns/k-tuples known?

I am looking for references regarding the following statement. For any two natural numbers x and y there must be a prime k-tuple (a, b, ...) corresponding to x consecutive primes (n+a, n+b, ...) for ...
Thomas Traill's user avatar
3 votes
1 answer
413 views

Primes from arithmetic and geometric progressions

The five primes, 131, 157, 211, 349, 739, are neither in arithmetic or geometric progression, but are instead the sum of the five corresponding terms of an arithmetic and geometric progression. Are ...
Bernardo Recamán Santos's user avatar
2 votes
1 answer
338 views

Is there a constant $\alpha$ such that: $P_{n+1} < P_n.\left(\frac{n+1}{n}\right)^\alpha$?

Is there a constant $\alpha$ such that: $$P_{n+1} < P_n \cdot \left(\frac{n+1}{n}\right)^\alpha$$ Or $$\lim_{n\to\infty}\frac{\ln\frac{P_{n+1}}{P_n}}{\ln\frac{n+1}{n}} < +\infty$$ Where $...
Đào Thanh Oai's user avatar
2 votes
2 answers
694 views

Summation involving Euler's totient function

Does the following sum have a closed-form expression? I've tried an Inclusion-Exclusion interpretation, to no avail: $f(n, p) = \sum_{i = 1} ^ n \lfloor \frac{n}{i} \rfloor \phi(p i) (-1) ^ i$ ($p$ ...
cupcake111680's user avatar
2 votes
1 answer
228 views

Equations involving arithmetic functions of primorials

Let $\sigma(n)=\sum_{1\leq d\mid n}d$ the sum of divisors, $\varphi(n)$ the Euler's totient function and we denote the primorial $\prod_{k=1}^n p_k$ as $N_n$, where $p_k$ denotes the $k$-th prime ...
user142929's user avatar
7 votes
0 answers
206 views

Does Morley's congruence characterize primes greater than $3$?

In 1895 Morley showed that $$\binom{p-1}{(p-1)/2}\equiv(-1)^{\frac{p-1}2}4^{p-1}\pmod{p^3}$$ for any prime $p>3$. In 2009, I formulated the following conjecture concerning the converse of Morley's ...
Zhi-Wei Sun's user avatar
  • 14.5k
3 votes
2 answers
333 views

For any integer $n>1$, there always exists at least one prime number $p$ with $n < p< n+\left(\ln\Big(\frac{n}{\ln n}\Big)+1\right)^2$

Question: Is the conjecture as follows true or false? For any integer $n>1$, there always exists at least one prime number $p$ with $$n < p< n+\left(\ln\Big(\frac{n}{\ln n}\...
Đào Thanh Oai's user avatar
10 votes
0 answers
263 views

On the infinity of $\{p\in \mathbb {N}:\exists n\in\mathbb{N}~p| \left \lfloor{r^n}\right \rfloor\}$

I've already asked this same question on MSE here, but didn't get much help, so I will try on this site as well. For which $r\in\mathbb{R}$ is the set $\mathscr{P}_r=\{p \in \mathbb{P}:\ (\exists n\...
Lucio Tanzini's user avatar
4 votes
1 answer
479 views

Are there infinite many two sided prime numbers?

A prime number $p=\overline{a_na_{n-1}\ldots a_1a_0}$ is called a two sided prime number if its reverse representation $q=\overline{a_0a_1\ldots a_{n-1}a_n}$ is a prime number too. Are there ...
Ali Taghavi's user avatar
0 votes
0 answers
118 views

At least a prime k-tuple

Let $k \in\mathbb{N}, k \geq 2$, and $\mathbb{P}$ represent the set of prime numbers. Consider the k-tuple $(0,h_1,h_2,\cdots,h_{k-1})$ with $0 < h_1 < \cdots < h_{k-1}$. The well known k-...
Lagrida Yassine's user avatar
6 votes
0 answers
200 views

some problems on sum of two squares

During my experiments with "Mathematica" I arrived to the following observations. My question is that are they interesting, known, solved or not. If they are known could you please give me a reference....
asad's user avatar
  • 841
-3 votes
1 answer
385 views

Published articles in journals about the Firoozbakht's conjecture, whose main goal or focus is the study of this conjecture

I would like to know what articles are in the literature about the known as Firoozbakht's conjecture, see the Wikipedia Firoozbakht's conjecture. Question. What articles have been published in ...
user142929's user avatar
2 votes
0 answers
186 views

Error bounds for $\pi(x)-Li(x)$ and convergence of the associated Dirichlet integral

Following On the integral $I_s =\int_{1}^{\infty} (\pi(x)-Li(x))x^{-s-1} dx$, define $$I_{s}=\int_{1}^{\infty} (\pi(x)-Li(x))x^{-s-1} \mathrm{d}x,$$ where $\pi$ denotes the prime counting function ...
Q_p's user avatar
  • 824
3 votes
0 answers
86 views

Are there infinitely many primes $p$, positive integers $ k, n $ such that $1 \le n < p$ and $p^k > n.rad(p^{k+1}−n)$?

Among $168$ prime numbers in range $1$ to $10^3$, there are $84$ prime numbers $n$ such that: $p^k> n.rad(p^{k+1}−n)$ where $1 \le n<p$ and $k=2,3,4$. There are also $84$ prime numbers $n$ such ...
Đào Thanh Oai's user avatar
4 votes
0 answers
919 views

Guessing of $n$th prime from "super- regularized" product of primes

( I've been thinking about asking this for a long time . Though this is not rigorous; It can be thought of as heuristic or extraction of information from different viewpoint.) We know "super-...
TPC's user avatar
  • 690
3 votes
1 answer
497 views

Generalization of Wilson's theorem for prime tuples

We know that Wilson's theorem states the following : $x$ is a prime if $(\frac {\Gamma(x)+1}{x})$ is an integer . We can extend this to Twin primes as : $x$ and $x+2$ is prime if $(\frac {4(\Gamma(...
TPC's user avatar
  • 690
5 votes
1 answer
197 views

applications of finding least quadratic nonresidue mod $p$?

I saw some papers from famous mathematicians (assuming GRH or without it) which are devoted to finding bound for least quadratic nonresidues modulo prime number $p$. My question is that why it is so ...
asad's user avatar
  • 841
1 vote
0 answers
62 views

On characterizations for Mersenne primes involving the sum of divisor function

In this post we denote the sum of positive divisors function of an integer $n\geq 1$ as $$\sigma(n)=\sum_{1\leq d\mid n}d.$$ Then a prime of the form $2^p-1$ is called a Mersenne prime. These are ...
user142929's user avatar
2 votes
0 answers
131 views

How many divisors of $\phi(m)$ do not divide $m-1$?

Lehmer's totient problem asks if there exists a composite number $m$ such that $\phi(m)$ divides $m-1$. Lower bounds on $m$ has been established but we do not know if a solution exists. Clearly, if we ...
Nilotpal Kanti Sinha's user avatar
9 votes
1 answer
962 views

A conjecture on primitive tenth roots of unity

QUESTION. How to solve my following conjecture involving primitive tenth roots of unity? Conjecture. Let $\zeta$ be any primitive tenth root of unity. Then $$\prod_{k=1}^{(p-1)/2}(\zeta-e^{2\pi ik^2/...
Zhi-Wei Sun's user avatar
  • 14.5k
3 votes
1 answer
206 views

Races that involve odd semiprimes: a first statement or conjecture

While I am studying the famous article [1], in English this is Andrew Granville and Greg Martin, Prime Number Races, The American Mathematical Monthly, vol. 113, (2006), I wondered what about a race ...
user142929's user avatar
1 vote
0 answers
111 views

Upper bound for $\alpha_{n}$ from Mertens' third theorem

This question is a follow-up to About Goldbach's conjecture. I would like to know if an unconditional upper bound for $\alpha_{n}$, defined as $n(N_{2}(n)-\dfrac{nN_{1}(n)}{P(n)})$ (where $N_{2}(...
Sylvain JULIEN's user avatar
2 votes
0 answers
99 views

A problem in modular roots

We have three mutually coprime integers $r,t,M$ where $M\asymp K^{\frac12-2\epsilon}$ and $r,t\asymp K^{\frac14+\epsilon}$ holds with some fixed $\epsilon>0$ and $K>0$ is a large parameter. ...
Turbo's user avatar
  • 13.7k
2 votes
1 answer
262 views

On a problem that equates $\frac{\text{prime}-1}{\operatorname{rad}(\text{prime}-1)}$ with the sequence of primorials

We denote for integers $m>1$ the product of the distinct prime numbers dividing $m$ as $$\operatorname{rad}(m)=\prod_{\substack{p\mid m\\p\text{ prime}}}p,$$ with the definition $\operatorname{rad}(...
user142929's user avatar
4 votes
0 answers
232 views

On the values of $\prod_{k=1}^{(p-1)/2}(e^{2\pi i/12}-e^{2\pi i k^2/p})$ for primes $p>3$

In a recent preprint, I investigated $$S_p(x):=\prod_{k=1}^{(p-1)/2}(x-e^{2\pi ik^2/p}),$$ where $p$ is an odd prime and $x$ is a root of unity. Motivated by Question 337879 and Question 338325, ...
Zhi-Wei Sun's user avatar
  • 14.5k
1 vote
1 answer
302 views

Number of prime factors and density

Let $\mathbb{N}$ denote the set of positive integers. For $n\in\mathbb{N}$ let $\mathbf{P}_n$ be the set of all positive integers $k$ such that there are at most $n$ different prime numbers that ...
Dominic van der Zypen's user avatar
2 votes
0 answers
192 views

A conjecture on crossing numbers related to primes

For a permutation $\sigma\in S_n$, its crossing number $\text{cr}(\sigma)$ is the number of pairs $\{i,j\}$ with $i,j\in\{1,\ldots,n\}$ such that $$i<j\le\sigma(i)<\sigma(j)\ \ \text{or}\ \ \...
Zhi-Wei Sun's user avatar
  • 14.5k
2 votes
0 answers
155 views

On the connection between $\pi(x)-Li(x)$ and $\theta(x)-x$

Let $\pi(x)$ be the number of primes $p$ not exceeding $x, \theta(x) = \sum_{p\leq x} \log p$ and $Li(x)$ be the logarithmic integral. Is it true that $$\pi(x)-Li(x) = \theta(x) - x + O(x^{1/2}\log^{...
Q_p's user avatar
  • 824
6 votes
0 answers
255 views

Convergence with the recurrence $T_{n+1}=T_n^2-T_n+\frac{n}{p_n}$

For each integer $n\geq 1$ I define the recurrence $$T_{n+1}=T_n^2-T_n+\frac{n}{p_n},$$ with $T_1=1$, where $p_k$ denotes the $k$-th prime. So multiplying by $(-1)^n$ and telescoping gives that for ...
user142929's user avatar
3 votes
0 answers
672 views

Prime numbers and sieving with $2,3,\cdots,q(x)= (1+o(1)) \log(x)$

Let $x \in \mathbb{R}_{+}$. For $q \in \mathbb{P}$, let : $\mathcal{B}_q = \{b \in \mathbb{N}^{*} \, | \, \gcd(b, \displaystyle{\small \prod_{\substack{p \leq q \\ \text{p prime}}} {\normalsize p}})=...
Lagrida Yassine's user avatar
3 votes
0 answers
236 views

Complexity of representations of sets using elementary functions

Fermat conjectured that $2^{2^n}+1$ is prime for every $n \in \mathbb{N}.$ Before even knowing about Euler's counterexample (that $2^{32}+1 = 641 \cdot 6700417$), you could possibly say that Fermat ...
Display name's user avatar
2 votes
0 answers
177 views

From Firoozbakht's conjecture to set interesting conjectures for sequences or series of primes

In this post we denote the $k-th$ prime number as $p_k$. I present two conjectures, the first about the asymptotic behaviour of a product and the other about an alternating series. I don't know if ...
user142929's user avatar
2 votes
2 answers
267 views

On values of $n\geq 1$ satisfying that for all primorial $N_k$ less than $n$ the difference $n-N_k$ is a prime number

In this post I present a similar question that shows section A19 of [1], I was inspired in it to define my sequence and question. I am asking about it since I think that the problem that arises from ...
user142929's user avatar
5 votes
0 answers
89 views

Is the ratio of a number to the variance of its divisors injective?

The variance $v_n$ of a natural number $n$ is defined as the variance of its divisors. There are distinct integer whose variances are equal e,g. $v_{691} = v_{817}$. However I observed that for $n \le ...
Nilotpal Kanti Sinha's user avatar
0 votes
1 answer
99 views

On the integral $I_s = \int_{1}^{\infty} (\pi(x)-Li(x))x^{-s-1} \mathrm{d}x$-follow up question

This is a follow up on On the integral $I_s =\int_{1}^{\infty} (\pi(x)-Li(x))x^{-s-1} dx$ According to the answer that i got, $I_s$ is not known to converge for any real $s<1$. But suppose $I_s$ ...
user avatar
2 votes
2 answers
428 views

On the integral $I_s =\int_{1}^{\infty} (\pi(x)-Li(x))x^{-s-1} dx$

Define $\pi(x)$ to be the prime counting function and Li(x) the logarithmic integral. Let $I_s$ be defined as above. Is $I_s$ known to be convergent for any real number $s<1$ ?
user avatar
0 votes
0 answers
83 views

Is it possible to get a conjecture similar to Mandl's conjecture for a different arithmetic function of number theory, mainly related to primes?

I'm curious to know if are in the literature analogous conjectures to the conjecture due to Mandl, I ask about these analogous conjectures for different sequences playing an important role in number ...
user142929's user avatar
4 votes
2 answers
679 views

On the product $\prod_{k=1}^{(p-1)/2}(x-e^{2\pi i k^2/p})$ with $x$ a root of unity

Let $p$ be an odd prime. Dirichlet's class number formula for quadratic fields essentially determines the value of the product $\prod_{k=1}^{(p-1)/2}(1-e^{2\pi ik^2/p})$. I think it is interesting to ...
Zhi-Wei Sun's user avatar
  • 14.5k
29 votes
2 answers
3k views

Is there a Kolmogorov complexity proof of the prime number theorem?

Lance Fortnow uses Kolmorogov complexity to prove an Almost Prime Number Theorem (https://lance.fortnow.com/papers/files/kaikoura.pdf, after theorem $2.1$): the $i$th prime is at most $i(\log i)^2$. ...
Turbo's user avatar
  • 13.7k
2 votes
0 answers
165 views

What about series involving strong primes?

I know about the importance in analytic number theory of the sutdy of series involving prime numbers or constellations of prime numbers, for example, if I am not wrong, major theorems are Mertens' ...
user142929's user avatar
9 votes
1 answer
445 views

A conjectural formula for the class number of the field $\mathbb Q(\sqrt{-p})$ with $p\equiv3\pmod8$

Question. Is my following conjecture new? How to prove it? Conjecture. Let $p>3$ be a prime with $p\equiv3\pmod 8$, and let $h(-p)$ denote the class number of the imaginary quadratic field $\...
Zhi-Wei Sun's user avatar
  • 14.5k
3 votes
0 answers
284 views

An attempt to get a variant of Agoh–Giuga conjecture

The idea of this post is an attempt to explore a variant of the so-called Agoh–Giuga conjecture. In past days, and today, I tried to think about variants of this conjecture exploring congruences about ...
user142929's user avatar
0 votes
1 answer
448 views

An upper bound for $\sqrt{p_{n+1}}$

Let $C$ be a positive constant. Is it true that for all sufficiently large integers $n$ the inequality $$\prod_{i=1}^n (1+\frac{1}{\sqrt{p_i}})>C\sqrt{p_{n+1}}$$ holds? (Here with $p_k$ is denoted ...
Kristiyan Vasilev's user avatar
2 votes
1 answer
321 views

Primes in shifted geometric sequence

Call a pair of integers $(a,b)$ trivial if it satisfies some simple divisibility condition, like for some prime $p$ we have $p$ divides both $a-1$ and $b+1$, or that $p$ divides both $a$ and $b$. This ...
domotorp's user avatar
  • 18.4k

1
13 14
15
16 17
41